Difference between revisions of "2018 AMC 12A Problems/Problem 24"

m (Solution 2 (Piecewise Function))
m (Solution 4 (Calculus))
(11 intermediate revisions by 3 users not shown)
Line 12: Line 12:
  
 
== Solution 1 (Expected Values) ==
 
== Solution 1 (Expected Values) ==
The expected value of Alice's number is <math>\frac12\left(1-0\right)=\frac12,</math> and the expected value of Bob's number is <math>\frac12\left(\frac23-\frac12\right)=\frac{7}{12}.</math> To maximize her chance of winning, Carol should choose the midpoint between these two expected values. So, the answer is <math>\frac12\left(\frac12+\frac{7}{12}\right)=\boxed{\textbf{(B) }\frac{13}{24}}.</math>
+
The expected value of Alice's number is <math>\frac12\left(0+1\right)=\frac12,</math> and the expected value of Bob's number is <math>\frac12\left(\frac12+\frac23\right)=\frac{7}{12}.</math> To maximize her chance of winning, Carol should choose the midpoint between these two expected values. So, the answer is <math>\frac12\left(\frac12+\frac{7}{12}\right)=\boxed{\textbf{(B) }\frac{13}{24}}.</math>
  
 
Alternatively, once we recognize that the answer lies in the interval <math>\left(\frac12,\frac{7}{12}\right),</math> we should choose <math>\textbf{(B)}</math> since no other answer choices lie in this interval.
 
Alternatively, once we recognize that the answer lies in the interval <math>\left(\frac12,\frac{7}{12}\right),</math> we should choose <math>\textbf{(B)}</math> since no other answer choices lie in this interval.
  
~Random_Guy (Solution)
+
~Random_Guy ~MRENTHUSIASM
 
 
~MRENTHUSIASM (Revision)
 
  
 
==Solution 2 (Piecewise Function)==
 
==Solution 2 (Piecewise Function)==
Line 42: Line 40:
 
<asy>
 
<asy>
 
/* Made by MRENTHUSIASM */
 
/* Made by MRENTHUSIASM */
size(220);
+
size(200);  
import TrigMacros;
 
 
 
label("$c$",(1.35,-0.1),UnFill);
 
label("$y$",(-0.1,1.35),UnFill);
 
  
 
real f(real x) { return x; }  
 
real f(real x) { return x; }  
Line 52: Line 46:
 
real h(real x) { return 1-x; }
 
real h(real x) { return 1-x; }
  
draw((1/2,-0.04)--(1/2,1.25),dashed);
+
draw((1/2,0)--(1/2,1.25),dashed);
draw((2/3,-0.04)--(2/3,1.25),dashed);
+
draw((2/3,0)--(2/3,1.25),dashed);
 
draw(graph(f,0,1/2),red);
 
draw(graph(f,0,1/2),red);
 
draw(graph(g,1/2,2/3),red);
 
draw(graph(g,1/2,2/3),red);
 
draw(graph(h,2/3,1),red);
 
draw(graph(h,2/3,1),red);
 +
 +
real xMin = -0.25;
 +
real xMax = 1.25;
 +
real yMin = -0.25;
 +
real yMax = 1.25;
 +
 +
draw((xMin,0)--(xMax,0),black+linewidth(1.5),EndArrow(5));
 +
draw((0,yMin)--(0,yMax),black+linewidth(1.5),EndArrow(5));
 +
label("$c$",(xMax,0),(2,0));
 +
label("$y$",(0,yMax),(0,2));
  
 
pair A[];
 
pair A[];
Line 73: Line 77:
 
label("$1$",(0,1),(-1.5,0));
 
label("$1$",(0,1),(-1.5,0));
  
rr_cartesian_axes(-0.25,1.25,-0.25,1.25,1.25,1.25,usegrid=false,useticks=true);
+
draw((1/2,-0.02)--(1/2,0.02),linewidth(1));
label("$\phantom{xxx}$",(1.6,-0.5),UnFill);
+
draw((2/3,-0.02)--(2/3,0.02),linewidth(1));
label("$\phantom{yyy}$",(-0.5,1.44),UnFill);
+
draw((1,-0.02)--(1,0.02),linewidth(1));
 +
draw((-0.02,1)--(0.02,1),linewidth(1));
 
</asy>
 
</asy>
 
Therefore, the maximum point of <math>P(c)</math> occurs in the interval <math>\left[\frac12,\frac23\right],</math> namely at <math>c=-\frac{13}{2\cdot(-12)}=\boxed{\textbf{(B) }\frac{13}{24}}.</math>
 
Therefore, the maximum point of <math>P(c)</math> occurs in the interval <math>\left[\frac12,\frac23\right],</math> namely at <math>c=-\frac{13}{2\cdot(-12)}=\boxed{\textbf{(B) }\frac{13}{24}}.</math>
Line 99: Line 104:
  
 
~MRENTHUSIASM
 
~MRENTHUSIASM
 +
 +
== Solution 4 (Calculus) ==
 +
There are two cases of winning:
 +
 +
Case 1: Alice choose a number that is smaller than Carol's, and Bob choose a number that is bigger.
 +
 +
Case 2: Alice choose a number that is bigger, and Bob choose a number that is smaller.
 +
 +
Let Carol's number be <math>\frac{1}{2}+x</math>, then the probability of Case 1 can be expressed by <math>\frac{1/2 + x}{1}\cdot\frac{1/3 - x }{1/3}</math>, and the probability of Case 2 can be expressed by <math>\frac{1/3 + 1/6 - x}{1}\cdot\frac{x}{1/3}</math>.
 +
 +
Then the probability of Carol winning can be expressed by the sum of the probability of Case 1 and the probability of Case 2: <math>3\left(\frac{1}{2} + x\right)\left(\frac{1}{3} - x\right)+ 3x\left(\frac{1}{2} - x\right)</math>, which simplify to <math>\frac{1}{2} + x - 6x^2</math>. Taking the first derivative, we get <math>1 - 12x</math>, which yield a maximum point at <math>x = \frac{1}{12}</math>. Hence, Carol should select <math>\frac{1}{2} + \frac{1}{12} = \boxed{\textbf{(B) }\frac{13}{24}}</math>.
  
 
== Video Solution by Richard Rusczyk ==
 
== Video Solution by Richard Rusczyk ==

Revision as of 19:40, 27 January 2023

Problem

Alice, Bob, and Carol play a game in which each of them chooses a real number between $0$ and $1.$ The winner of the game is the one whose number is between the numbers chosen by the other two players. Alice announces that she will choose her number uniformly at random from all the numbers between $0$ and $1,$ and Bob announces that he will choose his number uniformly at random from all the numbers between $\tfrac{1}{2}$ and $\tfrac{2}{3}.$ Armed with this information, what number should Carol choose to maximize her chance of winning?

$\textbf{(A) }\frac{1}{2}\qquad \textbf{(B) }\frac{13}{24} \qquad \textbf{(C) }\frac{7}{12} \qquad \textbf{(D) }\frac{5}{8} \qquad \textbf{(E) }\frac{2}{3}\qquad$

Solution 1 (Expected Values)

The expected value of Alice's number is $\frac12\left(0+1\right)=\frac12,$ and the expected value of Bob's number is $\frac12\left(\frac12+\frac23\right)=\frac{7}{12}.$ To maximize her chance of winning, Carol should choose the midpoint between these two expected values. So, the answer is $\frac12\left(\frac12+\frac{7}{12}\right)=\boxed{\textbf{(B) }\frac{13}{24}}.$

Alternatively, once we recognize that the answer lies in the interval $\left(\frac12,\frac{7}{12}\right),$ we should choose $\textbf{(B)}$ since no other answer choices lie in this interval.

~Random_Guy ~MRENTHUSIASM

Solution 2 (Piecewise Function)

Let $a,b,$ and $c$ be the numbers that Alice, Bob, and Carol choose, respectively.

Based on the value of $c,$ we construct the following table: \[\begin{array}{c|c|c} & & \\ [-2ex] \textbf{Case} & \textbf{Conditions for }\boldsymbol{a}\textbf{ and }\boldsymbol{b} & \textbf{Carol's Probability of Winning} \\ [0.5ex] \hline & & \\ [-1.5ex] 0<c<\frac12 & 0<a<c \text{ and } \frac12<b<\frac23 & \hspace{1.25mm}\frac{c}{1}\cdot\frac{1/6}{1/6}=c \\ [1.5ex] \frac12\leq c\leq\frac23 & \left(0<a<c \text{ and } c<b<\frac23\right) \text{ or } \left(c<a<1 \text{ and } \frac12<b<c\right) & \hspace{1.25mm}\frac{c}{1}\cdot\frac{2/3-c}{1/6}+\frac{1-c}{1}\cdot\frac{c-1/2}{1/6}=-12c^2+13c-3 \\ [1.5ex] \frac23<c<1 & c<a<1 \text{ and } \frac12<b<\frac23 & \hspace{4.375mm}\frac{1-c}{1}\cdot\frac{1/6}{1/6}=1-c \\ [1.5ex] \end{array}\] Let $P(c)$ be Carol's probability of winning when she chooses $c.$ We write $P(c)$ as a piecewise function: \[P(c) = \begin{cases} c & \mathrm{if} \ 0<c<\frac12 \\ -12c^2+13c-3 & \mathrm{if} \ \frac12\leq c\leq\frac23 \\ 1-c & \mathrm{if} \ \frac23<c<1 \end{cases}.\] Note that $P(c)$ is continuous in the interval $(0,1),$ increasing in the interval $\left(0,\frac12\right),$ increasing and then decreasing in the interval $\left(\frac12,\frac23\right),$ and decreasing in the interval $\left(\frac23,1\right).$ The graph of $y=P(c)$ is shown below. [asy] /* Made by MRENTHUSIASM */ size(200);   real f(real x) { return x; }  real g(real x) { return -12x^2+13x-3; } real h(real x) { return 1-x; }  draw((1/2,0)--(1/2,1.25),dashed); draw((2/3,0)--(2/3,1.25),dashed); draw(graph(f,0,1/2),red); draw(graph(g,1/2,2/3),red); draw(graph(h,2/3,1),red);  real xMin = -0.25; real xMax = 1.25; real yMin = -0.25; real yMax = 1.25;  draw((xMin,0)--(xMax,0),black+linewidth(1.5),EndArrow(5)); draw((0,yMin)--(0,yMax),black+linewidth(1.5),EndArrow(5)); label("$c$",(xMax,0),(2,0)); label("$y$",(0,yMax),(0,2));  pair A[]; A[0] = (0,0); A[1] = (1/2,1/2); A[2] = (2/3,1/3); A[3] = (1,0);  dot(A[1],red+linewidth(3.5));  dot(A[2],red+linewidth(3.5));   label("$0$",A[0],(-1.5,-1.5)); label("$\frac12$",(1/2,0),(0,-1.5)); label("$\frac23$",(2/3,0),(0,-1.5)); label("$1$",A[3],(0,-1.5)); label("$1$",(0,1),(-1.5,0));  draw((1/2,-0.02)--(1/2,0.02),linewidth(1)); draw((2/3,-0.02)--(2/3,0.02),linewidth(1)); draw((1,-0.02)--(1,0.02),linewidth(1)); draw((-0.02,1)--(0.02,1),linewidth(1)); [/asy] Therefore, the maximum point of $P(c)$ occurs in the interval $\left[\frac12,\frac23\right],$ namely at $c=-\frac{13}{2\cdot(-12)}=\boxed{\textbf{(B) }\frac{13}{24}}.$

~MRENTHUSIASM

Solution 3 (Answer Choices)

Let $a,b,$ and $c$ be the numbers that Alice, Bob, and Carol choose, respectively.

From the answer choices, we construct the following table: \[\begin{array}{c|c|c} & & \\ [-2ex] \boldsymbol{c} & \textbf{Conditions for }\boldsymbol{a}\textbf{ and }\boldsymbol{b} & \textbf{Carol's Probability of Winning} \\ [0.5ex] \hline & & \\ [-1.5ex] \frac12 & 0<a<\frac12 \text{ and } \frac12<b<\frac23 & \hspace{23.375mm}\frac{1/2}{1}\cdot\frac{1/6}{1/6}=\frac12 \\ [1.5ex] \frac{13}{24} & \left(0<a<\frac{13}{24} \text{ and } \frac{13}{24}<b<\frac23\right) \text{ or } \left(\frac{13}{24}<a<1 \text{ and } \frac12<b<\frac{13}{24}\right) & \frac{13/24}{1}\cdot\frac{1/8}{1/6}+\frac{11/24}{1}\cdot\frac{1/24}{1/6}=\frac{25}{48} \\ [1.5ex] \frac{7}{12} & \left(0<a<\frac{7}{12} \text{ and } \frac{7}{12}<b<\frac23\right) \text{ or } \left(\frac{7}{12}<a<1 \text{ and } \frac12<b<\frac{7}{12}\right) & \frac{7/12}{1}\cdot\frac{1/12}{1/6}+\frac{5/12}{1}\cdot\frac{1/12}{1/6}=\frac12 \\ [1.5ex] \frac58 & \left(0<a<\frac58 \text{ and } \frac58<b<\frac23\right) \text{ or } \left(\frac58<a<1 \text{ and } \frac12<b<\frac58\right) & \hspace{5.625mm}\frac{5/8}{1}\cdot\frac{1/24}{1/6}+\frac{3/8}{1}\cdot\frac{1/8}{1/6}=\frac{7}{16} \\ [1.5ex] \frac23 & \frac23<a<1 \text{ and } \frac12<b<\frac23 & \hspace{23.25mm}\frac{1/3}{1}\cdot\frac{1/6}{1/6}=\frac13 \\ [1.5ex] \end{array}\] Therefore, Carol should choose $\boxed{\textbf{(B) }\frac{13}{24}}$ to maximize her chance of winning.

~MRENTHUSIASM

Solution 4 (Calculus)

There are two cases of winning:

Case 1: Alice choose a number that is smaller than Carol's, and Bob choose a number that is bigger.

Case 2: Alice choose a number that is bigger, and Bob choose a number that is smaller.

Let Carol's number be $\frac{1}{2}+x$, then the probability of Case 1 can be expressed by $\frac{1/2 + x}{1}\cdot\frac{1/3 - x }{1/3}$, and the probability of Case 2 can be expressed by $\frac{1/3 + 1/6 - x}{1}\cdot\frac{x}{1/3}$.

Then the probability of Carol winning can be expressed by the sum of the probability of Case 1 and the probability of Case 2: $3\left(\frac{1}{2} + x\right)\left(\frac{1}{3} - x\right)+ 3x\left(\frac{1}{2} - x\right)$, which simplify to $\frac{1}{2} + x - 6x^2$. Taking the first derivative, we get $1 - 12x$, which yield a maximum point at $x = \frac{1}{12}$. Hence, Carol should select $\frac{1}{2} + \frac{1}{12} = \boxed{\textbf{(B) }\frac{13}{24}}$.

Video Solution by Richard Rusczyk

https://artofproblemsolving.com/videos/amc/2018amc12a/474

~ dolphin7

Video Solution (Meta-Solving Technique)

https://youtu.be/GmUWIXXf_uk?t=926

~ pi_is_3.14

See Also

2018 AMC 12A (ProblemsAnswer KeyResources)
Preceded by
Problem 23
Followed by
Problem 25
1 2 3 4 5 6 7 8 9 10 11 12 13 14 15 16 17 18 19 20 21 22 23 24 25
All AMC 12 Problems and Solutions

The problems on this page are copyrighted by the Mathematical Association of America's American Mathematics Competitions. AMC logo.png